Diễn Đàn MathScopeDiễn Đàn MathScope
  Diễn Đàn MathScope
Ghi Danh Hỏi/Ðáp Community Lịch

Go Back   Diễn Đàn MathScope > Sơ Cấp > Việt Nam và IMO > 2012

News & Announcements

Ngoài một số quy định đã được nêu trong phần Quy định của Ghi Danh , mọi người tranh thủ bỏ ra 5 phút để đọc thêm một số Quy định sau để khỏi bị treo nick ở MathScope nhé !

* Nội quy MathScope.Org

* Một số quy định chung !

* Quy định về việc viết bài trong diễn đàn MathScope

* Nếu bạn muốn gia nhập đội ngũ BQT thì vui lòng tham gia tại đây

* Những câu hỏi thường gặp

* Về việc viết bài trong Box Đại học và Sau đại học


Trả lời Gởi Ðề Tài Mới
 
Ðiều Chỉnh Xếp Bài
Old 22-02-2012, 06:35 PM   #1
namdung
Administrator

 
Tham gia ngày: Feb 2009
Đến từ: Tp Hồ Chí Minh
Bài gởi: 1,343
Thanks: 209
Thanked 4,066 Times in 778 Posts
Gửi tin nhắn qua Yahoo chát tới namdung
Trích:
Nguyên văn bởi Mashimaru View Post
Em tìm được kết quả là $S $ có thể có nhiều nhất $72 $ phần tử (không biết có đúng không ạ?) Lời giải dựa vào nhận xét rằng $S $ không thể chứa số $1 $ cũng như không thể chứa đồng thời 2 số nguyên tố $p, q $ với $10 < p, q < 100 $. Bây giờ từ $1 $ đến $100 $, bỏ đi $21 $ số nguyên tố lớn hơn $10 $, bỏ thêm số $1 $, ta có thể chia trường hợp để chứng minh S phải không chứa ít nhất $6 $ số trong các số còn lại này. Chứng minh không khó chỉ là có quá nhiều trường hợp nhỏ lẻ... Hy vọng TST không đố kiểu này
Đáp số bài này đúng là 72. Đặc điểm của các đề toán của TQ là xấu xí, gồ ghề và đôi khi phải rất là cơ bắp. Có lẽ cũng qua trường đào tạo của các đề toán này mà học sinh TQ rất lì đòn, bài nào cũng tìm cách chiến đấu được.
------------------------------
Trích:
Nguyên văn bởi chemthan View Post
*) Trường hợp $a=203 $.
Khi đó $203^m+1|203^{n-1}+1 $.
Suy ra $(a,m,n)=(203,k,(2l+1)k+1) $.
*) Trường hợp $a\ne 203 $.
Đặt $n=km+r $, trong đó $k,r\in N; 0\le r<m $.
Ta có $a^m+1|(-1)^k.a^r+203 $.
Vì $a\ne 203 $ nên $(-1)^k.a^r+203\ne 0 $ $\Rightarrow a^m+1\le |(-1)^k.a^r+203|\le a^{m-1}+203 $.
$\Rightarrow a^{m-1}(a-1)\le 202 $.
$\Rightarrow m\le 8 $.
Đến đây xét các trường hợp của $m $ là ra.
Tương tự như trên, với đề TQ thì giải dài là chuyện bình thường.
[RIGHT][I][B]Nguồn: MathScope.ORG[/B][/I][/RIGHT]
 
namdung is offline   Trả Lời Với Trích Dẫn
Old 22-02-2012, 08:52 PM   #2
namdung
Administrator

 
Tham gia ngày: Feb 2009
Đến từ: Tp Hồ Chí Minh
Bài gởi: 1,343
Thanks: 209
Thanked 4,066 Times in 778 Posts
Gửi tin nhắn qua Yahoo chát tới namdung
Vì chưa có kết quả VMO nên chưa có bạn thí sinh nào tự tin vào chủ đề này, do đó dường như chủ đề vẫn chỉ là nơi độc diễn của các cựu VMO, TST, IMO.

Tôi tiếp tục đưa một số bài tập mới

(Trích từ đề thi Ba Lan, 2012)

9. (Trung bình): Tìm tất cả các hàm số $f, g: R --> R $ thỏa
$g(f(x) - y) = f(g(y)) + x $ với mọi x, y thuộc R.

10. (Trung bình khó) Cho tam giác ABC có $A=60^0 $ và AB≠AC. Gọi I,O lần lượt là tâm đường tròn nội và ngoại tiếp tam giác. Chứng minh rằng: trung trực của AI, đường thẳng OI và đường thằng BC đồng quy.

11. (Khó) Giả sử m,n là các số nguyên dương sao cho tập hợp A={1,2,...,n} có đúng m số nguyên tố khác nhau. Chứng minh rằng nếu ta chọn bất kì m+1 số khác nhau từ A thì ta có thể tìm 1 số từ m+1 số đã chọn chia hết tích m số khác.

Có 2 bài hình học, mời các bạn tham gia phân tích tìm tòi cách giải.
[RIGHT][I][B]Nguồn: MathScope.ORG[/B][/I][/RIGHT]
 
namdung is offline   Trả Lời Với Trích Dẫn
The Following User Says Thank You to namdung For This Useful Post:
huynhcongbang (29-02-2012)
Old 21-02-2012, 02:05 AM   #3
huynhcongbang
Administrator

 
huynhcongbang's Avatar
 
Tham gia ngày: Feb 2009
Đến từ: Ho Chi Minh City
Bài gởi: 2,413
Thanks: 2,165
Thanked 4,188 Times in 1,381 Posts
Gửi tin nhắn qua Yahoo chát tới huynhcongbang
Trích:
Nguyên văn bởi Nguyenhuyen_AG View Post
Đây cũng là một bài toán rất thú vị.
Bài 6: (Làm mạnh Việt Nam TST 1996) Với $a,b,c$ là các số thực bất kỳ, khi đó ta luôn có $$(a+b)^4+(b+c)^4+(c+a)^4\ge \frac{4}{7}[a^4+b^4+c^4+(a+b+c)^4].$$
Bài này giải rất nhẹ nhàng!
Đặt $a=\frac{y+z-x}{2}, b= \frac{z+x-y}{2},c = \frac{x+y-z}{2} $.
Thay vào BĐT cần chứng minh và rút gọn, ta được:
$x^4+y^4+z^4 \ge x^2y^2+y^2z^2+z^2x^2 $.
BĐT cuối hiển nhiên đúng nên ta có đpcm.

Bài TST 1996 cũng được giải theo cách tương tự, không cần phải dồn biến hay xét hàm số gì cả.
------------------------------
Sẵn dịp nói về thi TST, mình muốn trao đổi với mọi người bài tổ hợp số 6 trong kì thi VN TST 1990 với nội dung như sau:

Cho n học sinh ($n \ge 3 $) đứng thành một vòng tròn và quay mặt vào cô giáo ở tâm đường tròn. Mỗi lần cô giáo thổi còi thì hai em nào đó đứng cạnh nhau đổi chỗ cho nhau, các em còn lại đứng im. Tìm số M nhỏ nhất sao cho sau M lần thổi còi, bằng cách đổi chỗ như trên một cách thích hợp thì các em học sinh đứng được thành một vòng tròn mới sao cho: Hai em lúc đầu đứng cạnh nhau, nhưng hai em đó, tạm gọi là A và B, thì nếu lúc đầu A đứng bên tay trái của B thì lúc kết thúc, A đứng bên tay phải của B.

Trên thực tế, bài này có thể phát biểu ở một dạng khác dễ hiểu hơn mà mình đã nhờ bạn hoanghai_vovn post giúp:

Trên một đường đua khép kín, có n chiếc xe đang chạy với một thứ tự nhất định theo chiều kim đồng hồ. Tại mỗi thời điểm, có đúng một chiếc xe tăng tốc và vượt qua mặt chiếc xe ngay trước nó (tức là hai chiếc xe liền nhau đó đổi chỗ cho nhau).
Hỏi sau ít nhất bao nhiều lần vượt thì các chiếc xe sẽ vẫn với thứ tự cũ trên đường đua nhưng theo ngược chiều kim đồng hồ?

Theo các thảo luận bên math.vn thì mình đã dự đoán được có thể là công thức sau:
$f(n) = \sum_{i=1}^{n-1}\left [ \frac{i-1}{2} \right ] = \sum_{i=1}^{n} \left (n - 2\left [ \frac{i}{2} \right ] \right ) $. (*)
trong đó $f(n) $ chính là số cần tìm ứng với trường hợp n xe.
Một số kết quả khác như:
+ $f(3)=1, f(4)=2, f(5)=4, f(6)=6,... $
+ Do việc vượt qua mặt của các chiếc xe có thể xem như chúng đổi vị trí cho nhau nên có thể xem 1 xe nào đó, đặt là xe 1, không thay đổi trong quá trình sắp xếp. Giả sử các xe có thứ tự là $1, 2, 3, ..., n-1, n $ theo chiều kim đồng hồ. Ta cần đạt được thứ tự đó lần nữa nhưng ngược chiều kim đồng hồ.
+ Nếu gọi S(i) là tổng số nghịch thế (số nhỏ hơn nhưng lại nằm sau theo chiều kim đồng hồ) tính từ một vị trí bất kì trên vòng tròn đến cuối vòng (ngay trước xe số 1) trong lần đổi chỗ thứ i thì:
$S(0)=0, S(f(n))=\frac{(n-1)(n-2)}{2} $ và đây là số nghịch thế lớn nhất.

Đó là một số kết quả mình tìm hiểu được và nếu các công thức (*) đúng thì hoàn toàn có thể phát biểu được các bước di chuyển. Tuy nhiên, quan trọng là điều kiện cần vẫn chưa có, chưa chứng minh được giá trị kia cũng là nhỏ nhất!
Mong mọi người cùng thảo luận.
[RIGHT][I][B]Nguồn: MathScope.ORG[/B][/I][/RIGHT]
 
__________________
Sự im lặng của bầy mèo
huynhcongbang is offline   Trả Lời Với Trích Dẫn
The Following User Says Thank You to huynhcongbang For This Useful Post:
pco (03-03-2012)
Old 19-02-2012, 09:15 PM   #4
hotraitim
+Thành Viên+
 
Tham gia ngày: Dec 2011
Bài gởi: 41
Thanks: 79
Thanked 8 Times in 5 Posts
Chứng minh với mọi số thực dương a,b,c thì:
$a^{3}+b^{3}+c^{3}+3abc\geq \sum ab\sqrt{2\left ( a^{2}+b^{2} \right )} $
[RIGHT][I][B]Nguồn: MathScope.ORG[/B][/I][/RIGHT]
 
hotraitim is offline   Trả Lời Với Trích Dẫn
Old 19-02-2012, 09:19 PM   #5
5434
+Thành Viên+
 
5434's Avatar
 
Tham gia ngày: Sep 2011
Đến từ: no*i ty bă't đâ'u
Bài gởi: 695
Thanks: 121
Thanked 335 Times in 214 Posts
Trích:
Nguyên văn bởi hotraitim View Post
Chứng minh với mọi số thực dương a,b,c thì:
$a^{3}+b^{3}+c^{3}+3abc\geq \sum ab\sqrt{2\left ( a^{2}+b^{2} \right )} $
Bài nầy có 1 cách là xài SOS
[RIGHT][I][B]Nguồn: MathScope.ORG[/B][/I][/RIGHT]
 
__________________

5434 is offline   Trả Lời Với Trích Dẫn
The Following User Says Thank You to 5434 For This Useful Post:
Trầm (05-03-2012)
Old 01-03-2012, 10:29 PM   #6
huynhcongbang
Administrator

 
huynhcongbang's Avatar
 
Tham gia ngày: Feb 2009
Đến từ: Ho Chi Minh City
Bài gởi: 2,413
Thanks: 2,165
Thanked 4,188 Times in 1,381 Posts
Gửi tin nhắn qua Yahoo chát tới huynhcongbang
Cách của bạn Hải dành cho bài 15 tuyệt thật, không ngờ lại có thể biến đổi trực tiếp theo độ dài cạnh của tam giác như thế.
Dưới đây là cách dùng $p, R, r$ thường gặp đối với các bài toán dạng này:

Gọi $p$ là nửa chu vi của tam giác $ABC$, vì tam giác $ABC$ nhọn nên $0<x,y,z<1$.
Trước tiên ta sẽ chứng minh các kết quả sau :
1) $\cos A,\cos B,\cos C$ là nghiệm của phương trình :
\[4{{R}^{2}}{{X}^{3}}-4R(R+r){{X}^{2}}+({{p}^{2}}+{{r}^{2}}-4{{R}^{2}})X+{{(2R+r)}^{2}}-{{p}^{2}}=0 (*) \]
2) ${{p}^{2}}\le 4{{R}^{2}}+3{{r}^{2}}+4Rr$ ( Bất đẳng thức Gerretsen )
*Chứng minh:
1) Gọi $a,b,c$ lần lượt là ba cạnh của tam giác $ABC$, khi đó ta có :
\[a=2R\sin A=2R\sqrt{1-{{\cos }^{2}}A}\] và \[p-a=r\cot \frac{A}{2}=r\sqrt{\frac{1+\cos A}{1-\cos A}}\] nên ta có :
\[p=a+(p-a)=2R\sqrt{(1-\cos A)(1+\cos A)}+r\sqrt{\frac{1+\cos A}{1-\cos A}}\] \[\Leftrightarrow {{p}^{2}}=4{{R}^{2}}(1-\cos A)(1+\cos A)+4Rr(1+\cos A)+{{r}^{2}}\frac{1+\cos A}{1-\cos A}\]
\[\Leftrightarrow {{p}^{2}}(1-\cos A)=4{{R}^{2}}{{(1-\cos A)}^{2}}(1+\cos A)+4Rr(1-{{\cos }^{2}}A)+{{r}^{2}}(1+\cos A)\] \[\Leftrightarrow 4{{R}^{2}}{{\cos }^{3}}A-4R(R+r){{\cos }^{2}}A+({{p}^{2}}+{{r}^{2}}-4{{R}^{2}})\cos A+{{(2R+r)}^{2}}-{{p}^{2}}=0\]
Do đó, $\cos A$ là một nghiệm của phương trình (*).
Hoàn toàn tương tự ta cũng có $\cos B,\cos C$ là nghiệm của phương trình (*).
Vì (*) là một phương trình bậc ba nên $\cos A,\cos B,\cos C$ là 3 nghiệm của nó và 1) được chứng minh.
Theo 1) thì $\cos A,\cos B,\cos C$ là ba nghiệm của phương trình : \[4{{R}^{2}}{{X}^{3}}-4R(R+r){{X}^{2}}+({{p}^{2}}+{{r}^{2}}-4{{R}^{2}})X+{{(2R+r)}^{2}}-{{p}^{2}}=0\] nên theo định lý Vi-ét thì : \[\cos A+\cos B+\cos C=-\frac{-4R(R+r)}{4{{R}^{2}}}=\frac{R+r}{R}\]
Cũng theo 1) thì $\cos A,\cos B,\cos C$ là ba nghiệm của phương trình : \[4{{R}^{2}}{{X}^{3}}-4R(R+r){{X}^{2}}+({{p}^{2}}+{{r}^{2}}-4{{R}^{2}})X+{{(2R+r)}^{2}}-{{p}^{2}}=0\] nên theo định lý Vi-ét thì : \[\cos A.\cos B.\cos C=-\frac{{{(2R+r)}^{2}}-{{p}^{2}}}{4{{R}^{2}}}=\frac{{{p}^{2}}-4{{R}^{2}}-{{r}^{2}}-4Rr}{4{{R}^{2}}}\]
Để cho gọn ta kí hiệu: \[\sum{{{\cos }^{2}}A(\cos B+\cos C)={{\cos }^{2}}A(\cos B+\cos C)+{{\cos }^{2}}B(\cos C+\cos A)+{{\cos }^{2}}C(\cos A+\cos B)}\]
Theo 1) thì $\cos A,\cos B,\cos C$ là ba nghiệm của phương trình : \[4{{R}^{2}}{{X}^{3}}-4R(R+r){{X}^{2}}+({{p}^{2}}+{{r}^{2}}-4{{R}^{2}})X+{{(2R+r)}^{2}}-{{p}^{2}}=0\] nên theo định lý Vi-ét thì :
\[\cos A\cos B+\cos B\cos C+\cos C\cos A=\frac{{{p}^{2}}+{{r}^{2}}-4{{R}^{2}}}{4{{R}^{2}}}\]
Ta có : \[\begin{align}
& (\cos A+\cos B)(\cos B+\cos C)(\cos C+\cos A)=(\frac{R+r}{R}-\cos A)(\frac{R+r}{R}-\cos B)(\frac{R+r}{R}-\cos C) \\
& =(\frac{R+r}{R}-\cos A)\left[ {{(\frac{R+r}{R})}^{2}}-(\frac{R+r}{R})\cos B-(\frac{R+r}{R})\cos C+\cos B\cos C \right] \\
& =(\frac{R+r}{R})(\cos A\cos B+\cos B\cos C+\cos C\cos A)-\cos A\cos B\cos C \\
& =(\frac{R+r}{R}).(\frac{{{p}^{2}}+{{r}^{2}}-4{{R}^{2}}}{4{{R}^{2}}})-\frac{{{p}^{2}}-4{{R}^{2}}-{{r}^{2}}-4Rr}{4{{R}^{2}}}=\frac{r.(2Rr+{{p}^{2}}+{{r}^{2}}) }{4{{R}^{3}}} \\
\end{align}\]
Mà \[(\cos A+\cos B)(\cos B+\cos C)(\cos C+\cos A)=\sum{{{\cos }^{2}}A(\cos B+\cos C)+2\cos A\cos B\cos C}\]
\[=\sum{{}}{{\cos }^{2}}A(\cos B+\cos C)+2.\frac{{{p}^{2}}-4{{R}^{2}}-{{r}^{2}}-4Rr}{4{{R}^{2}}}\]
Suy ra \[\sum{{{\cos }^{2}}A(\cos B+\cos C)=\frac{r(2Rr+{{p}^{2}}+{{r}^{2}})}{4{{R}^{3}}}}-2.\frac{{{p}^{2}}-4{{R}^{2}}-{{r}^{2}}-4Rr}{4{{R}^{2}}}\]
\[=\frac{2R{{r}^{2}}+{{p}^{2}}r+{{r}^{3}}-2R{{p}^{2}}+8{{R}^{3}}+2{{r}^{2}}R+8{{R}^{2}}p}{4{ {R}^{3}}}=\frac{2R{{r}^{2}}+{{p}^{2}}r+{{r}^{3}}-2R{{p}^{2}}+8{{R}^{3}}+8{{R}^{2}}r+2R{{r}^{2}}}{4{ {R}^{3}}}\]
Trở lại bài toán , ta cần chứng minh rằng : \[\max \left\{ \frac{x}{y}+\frac{y}{x}+2;\frac{y}{z}+\frac{z}{y}+ 2;\frac{z}{x}+\frac{x}{z}+2 \right\}\ge \frac{2}{x+y+z-1}\]
Giả sử ngược lại là: \[\frac{x}{y}+\frac{y}{x}+2<\frac{2}{x+y+z-1};\quad \frac{y}{z}+\frac{z}{y}+2<\frac{2}{x+y+z-1};\quad \frac{z}{x}+\frac{x}{z}<\frac{2}{x+y+z-1}\]
Ta có :
\[\begin{align}
& \frac{x}{y}+\frac{y}{x}+2<\frac{2}{x+y+z-1}\Leftrightarrow (x+y+z-1)({{x}^{2}}+{{y}^{2}}+2xy)<2xy \\
& \Leftrightarrow {{x}^{3}}+x{{y}^{2}}+2{{x}^{2}}y+{{x}^{2}}y+{{y}^{ 3}}+2x{{y}^{2}}+z{{x}^{2}}+z{{y}^{2}}+2xyz-{{x}^{2}}-{{y}^{2}}-2xy<2xy \\
& \Leftrightarrow {{x}^{3}}+{{y}^{3}}+3{{x}^{2}}y+3x{{y}^{2}}+{{x}^{ 2}}z+{{y}^{2}}z<4xy+{{x}^{2}}+{{y}^{2}} \\
\end{align}\]
Tương tự ta có :
\[\begin{align}
& {{y}^{3}}+{{z}^{3}}+3{{y}^{2}}z+3y{{z}^{2}}+{{z}^{ 2}}x+{{y}^{2}}x<4yz+{{y}^{2}}+{{z}^{2}};\ \\
& {{z}^{3}}+{{x}^{3}}+3{{z}^{2}}x+3z{{x}^{2}}+{{z}^{ 2}}y+{{x}^{2}}y<2xz+{{x}^{2}}+{{z}^{2}} \\
\end{align}\]
Cộng ba bất đẳng thức trên vế theo vế ta được:
\[2({{x}^{3}}+{{y}^{3}}+{{z}^{3}})+4[{{x}^{2}}(y+z)+{{y}^{2}}(z+x)+{{z}^{2}}(x+y)]<4(xy+yz+zx)+2({{x}^{2}}+{{y}^{2}}+{{z}^{2}})\]
\[\Leftrightarrow {{(x+y+z)}^{3}}<{{(x+y+z)}^{2}}+3xyz+{{x}^{2}}(y+z )+{{y}^{2}}(z+x)+{{z}^{2}}(x+y)\]
Đến đây áp dụng các kết quả chứng minh ở trên , ta có :
\[\begin{align}
& {{\left( \frac{R+r}{R} \right)}^{3}}<{{\left( \frac{R+r}{R} \right)}^{2}}+3.\left( \frac{{{p}^{2}}-4{{R}^{2}}-4Rr-{{r}^{2}}}{4{{R}^{2}}} \right)+\frac{2R{{r}^{2}}+{{p}^{2}}r+{{r}^{3}}-2R{{p}^{2}}+8{{R}^{3}}+8{{R}^{2}}r+2R{{r}^{2}}}{4{ {R}^{3}}} \\
& \Leftrightarrow 4{{R}^{3}}+12{{R}^{2}}r+12R{{r}^{2}}+4{{r}^{3}}<(4 {{R}^{3}}+4R{{r}^{2}}+8{{R}^{2}}r)+(3R{{p}^{2}}-12{{R}^{3}}-12{{R}^{2}}r-3R{{r}^{2}})+ \\
& (8{{R}^{3}}+{{r}^{3}}+8{{R}^{2}}r+4R{{r}^{2}}+{{p} ^{2}}r-2R{{p}^{2}}) \\
& \Leftrightarrow 4{{R}^{3}}+3{{r}^{3}}+8{{R}^{2}}r+7R{{r}^{2}}<{{p} ^{2}}(R+r) \\
& \Leftrightarrow (4{{R}^{3}}+4{{R}^{2}}r)+(3{{r}^{3}}+3R{{r}^{2}})+ (4{{R}^{2}}r+4R{{r}^{2}})<{{p}^{2}}(R+r) \\
& \Leftrightarrow (R+r)(4{{R}^{2}}+3{{r}^{2}}+4Rr)<{{p}^{2}}(R+r) \\
& \Leftrightarrow 4{{R}^{2}}+3{{r}^{2}}+4Rr<{{p}^{2}} \\
\end{align}\]
Trái với kết quả 2). Do đó điều giả sử là sai, tức là ta phải có :
\[\max \left\{ \frac{x}{y}+\frac{y}{x}+2;\frac{y}{z}+\frac{z}{y}+ 2;\frac{z}{x}+\frac{x}{z}+2 \right\}\ge \frac{2}{x+y+z-1}\]
Đây cũng chính là đpcm.
[RIGHT][I][B]Nguồn: MathScope.ORG[/B][/I][/RIGHT]
 
__________________
Sự im lặng của bầy mèo
huynhcongbang is offline   Trả Lời Với Trích Dẫn
The Following 2 Users Say Thank You to huynhcongbang For This Useful Post:
pco (03-03-2012), thinhptnk (03-03-2012)
Old 19-02-2012, 09:20 PM   #7
nghiepdu-socap
+Thành Viên+
 
nghiepdu-socap's Avatar
 
Tham gia ngày: Apr 2010
Bài gởi: 193
Thanks: 195
Thanked 129 Times in 72 Posts
Giải bài 5:
Ta sẽ chứng minh nhận xét sau: Trong 2 số $a_i,b_i $ 1 số >n, 1 số <n+1
Thật vậy: giả sử tồn tại i sao cho $a_i,b_i < n+1 $
Suy ra $n+1>a_i>a_{i-1}>...>a_1; n+1>b_i>b_{i+1}>...>b_n $
Như vậy sẽ có n+1 số <n+1 (vô lý)
Từ nhận xét dễ có điều phải chứng minh
[RIGHT][I][B]Nguồn: MathScope.ORG[/B][/I][/RIGHT]
 
nghiepdu-socap is offline   Trả Lời Với Trích Dẫn
Old 19-02-2012, 10:24 PM   #8
namdung
Administrator

 
Tham gia ngày: Feb 2009
Đến từ: Tp Hồ Chí Minh
Bài gởi: 1,343
Thanks: 209
Thanked 4,066 Times in 778 Posts
Gửi tin nhắn qua Yahoo chát tới namdung
Bài 3 đúng là có 1 cách dựa vào định lý Turan (đồ thị không chứa đồ thị con đầy đủ bậc 4) và nhận xét cơ bản: Giữa 4 thành phố có nhiều nhất 5 con đường.

Cũng nhắc lại rằng khi sử dụng định lý Turan ta phải chứng minh. (Bằng quy nạp như Mashimaru đã nói).

Có 1 cách khác giải trực tiếp như sau (lời giải do Nguyễn Mạnh Tiến, SV năm 2 ĐHKHTN Tp HCM đề xuất):

Gọi A là thành phố có nhiều đường đi nhất (gồm cả đường đi xuất phát từ A và đường đi đến A). Ta chia các thành phố còn lại thành 3 loại. Loại I - Có đường đi xuất phát từ A. Loại II - Có đường đi đến A. Loại III: Không có đường đi đến A hoặc xuất phát từ A. Đặt m = | I |, n = | II |, p = | III |. Ta có m + n + p = 209.

Dễ thấy giữa các thành phố loại I không có đường đi. Tương tự, giữa các thành phố loại 2 không có đường đi.

Số các đường đi liên quan đến các thành phố loại 3 không vượt quá p(m+n). (Do bậc của A = m + n là lớn nhất).

Tổng số đường đi bao gồm:
+ Các đường đi liên quan đến A: m + n
+ Các đường đi liên quan đến III : $\le p(m+n) $
+ Các đường đi giữa I và II: $\le mn $

Suy ra tổng số đường đi nhỏ hơn $mn + (p+1)m+ (p+1)n \le (m+n+p+1)^2/3 = 210^2/3 $.

Dấu bằng xảy ra với đồ thị 3 phe, mỗi phe có 70 thành phố, thành phố phe 1 có đường đi đến thành phố phe 2, thành phố phe 2 có đường đi đến thành phố phe 3, thành phố phe 3 có đường đi đến thành phố phe 1 (như Traum và Mashimaru đã chỉ ra).
[RIGHT][I][B]Nguồn: MathScope.ORG[/B][/I][/RIGHT]
 
namdung is offline   Trả Lời Với Trích Dẫn
The Following User Says Thank You to namdung For This Useful Post:
hahahaha4 (08-03-2012)
Old 22-02-2012, 05:24 PM   #9
chemthan
Administrator

 
chemthan's Avatar
 
Tham gia ngày: Mar 2009
Bài gởi: 349
Thanks: 0
Thanked 308 Times in 161 Posts
Trích:
Nguyên văn bởi namdung View Post
Bài 8. Tìm tất cả các bộ số nguyên dương (a, m, n) sao cho:
(1) a > 1, m > 1;
(2) $a^m + 1 | a^n + 203 $.
*) Trường hợp $a=203 $.
Khi đó $203^m+1|203^{n-1}+1 $.
Suy ra $(a,m,n)=(203,k,(2l+1)k+1) $.
*) Trường hợp $a\ne 203 $.
Đặt $n=km+r $, trong đó $k,r\in N; 0\le r<m $.
Ta có $a^m+1|(-1)^k.a^r+203 $.
Vì $a\ne 203 $ nên $(-1)^k.a^r+203\ne 0 $ $\Rightarrow a^m+1\le |(-1)^k.a^r+203|\le a^{m-1}+203 $.
$\Rightarrow a^{m-1}(a-1)\le 202 $.
$\Rightarrow m\le 8 $.
Đến đây xét các trường hợp của $m $ là ra.
[RIGHT][I][B]Nguồn: MathScope.ORG[/B][/I][/RIGHT]
 
chemthan is offline   Trả Lời Với Trích Dẫn
Old 22-02-2012, 09:39 PM   #10
hotraitim
+Thành Viên+
 
Tham gia ngày: Dec 2011
Bài gởi: 41
Thanks: 79
Thanked 8 Times in 5 Posts
Bài 7:CHứng minh với mọi a,b,c không âm thì:
$a^{3}+b^{3}+c^{3}+3abc\geq \sum ab\sqrt{2\left ( a^{2}+b^{2} \right )} $
[RIGHT][I][B]Nguồn: MathScope.ORG[/B][/I][/RIGHT]
 
hotraitim is offline   Trả Lời Với Trích Dẫn
Old 22-02-2012, 09:58 PM   #11
tranghieu95
+Thành Viên+
 
tranghieu95's Avatar
 
Tham gia ngày: Oct 2010
Đến từ: THPT Phan Bội Châu- Nghệ An
Bài gởi: 382
Thanks: 187
Thanked 364 Times in 197 Posts
Gửi tin nhắn qua Yahoo chát tới tranghieu95
Trích:
Nguyên văn bởi hotraitim View Post
Bài 7:CHứng minh với mọi a,b,c không âm thì:
$a^{3}+b^{3}+c^{3}+3abc\geq \sum ab\sqrt{2\left ( a^{2}+b^{2} \right )} $
Bài này có ở đây rồi mà bạn
[Only registered and activated users can see links. ]
[RIGHT][I][B]Nguồn: MathScope.ORG[/B][/I][/RIGHT]
 
__________________
TỪ TỪ LÀ HẠNH PHÚC
A1K39
XIN LỖI ĐÃ THẤT HỨA NHÉ

KỆ
tranghieu95 is offline   Trả Lời Với Trích Dẫn
Old 22-02-2012, 11:00 PM   #12
namdung
Administrator

 
Tham gia ngày: Feb 2009
Đến từ: Tp Hồ Chí Minh
Bài gởi: 1,343
Thanks: 209
Thanked 4,066 Times in 778 Posts
Gửi tin nhắn qua Yahoo chát tới namdung
Trích:
Nguyên văn bởi hotraitim View Post
Bài 7:CHứng minh với mọi a,b,c không âm thì:
$a^{3}+b^{3}+c^{3}+3abc\geq \sum ab\sqrt{2\left ( a^{2}+b^{2} \right )} $
Bài này xuất hiện lần đầu tiên trong một tài liệu của Trần Tuấn Anh. Ý tưởng cơ bản là dựa vào đánh giá thú vị sau:

$\sqrt{2(a^2+b^2)} - (a+b) = \frac{2(a^2+b^2)-(a+b)^2}{\sqrt{2(a^2+b^2)} + (a+b)} = \frac{(a-b)^2}{\sqrt{2(a^2+b^2)} + (a+b)} \le \frac{(a-b)^2}{2(a+b)}
$

Từ đó suy ra $\sqrt{2(a^2+b^2)} \le a+b + \frac{(a-b)^2}{2(a+b)} $
[RIGHT][I][B]Nguồn: MathScope.ORG[/B][/I][/RIGHT]
 
namdung is offline   Trả Lời Với Trích Dẫn
Old 22-02-2012, 11:05 PM   #13
cleverboy
+Thành Viên+
 
Tham gia ngày: Dec 2008
Bài gởi: 108
Thanks: 17
Thanked 58 Times in 32 Posts
Em xin giải bài 10. Em đánh vội nên nếu khó đọc thì mong thầy thông cảm cho em!
[RIGHT][I][B]Nguồn: MathScope.ORG[/B][/I][/RIGHT]
 
File Kèm Theo
Kiểu File : pdf bài tập 10.pdf (165.4 KB, 75 lần tải)
cleverboy is offline   Trả Lời Với Trích Dẫn
Old 23-02-2012, 09:39 PM   #14
kien10a1
+Thành Viên+
 
kien10a1's Avatar
 
Tham gia ngày: Feb 2011
Đến từ: Vĩnh Yên- Vĩnh Phúc
Bài gởi: 371
Thanks: 43
Thanked 263 Times in 153 Posts
Gửi tin nhắn qua Yahoo chát tới kien10a1
Trích:
Nguyên văn bởi cleverboy View Post
Em xin giải bài 10. Em đánh vội nên nếu khó đọc thì mong thầy thông cảm cho em!
Em nghĩ có cách đơn giản hơn

Gọi K là trung điểm AI thì K thuộc (I), tiếp tuyến KS của (I) với S trên BC là trung trực AI, SK cắt lại cung AB không chứa C tại P thì C,I, P thẳng hàng.
vẽ SI cắt (CPS) tại O', suy ra O'P=O'C, và $\widehat{SO'C}= \widehat{SPC}=\widehat{NBC } $ nên O'ICB nội tiếp, dễ thấy O cũng thuộc trung trực PC, và $\widehat{BOC}=2\widehat{BAC}=120=\widehat{BIC} $ , nên O trùng O', suy ra ĐPCM
[RIGHT][I][B]Nguồn: MathScope.ORG[/B][/I][/RIGHT]
 
__________________
Quay về với nơi bắt đầu
kien10a1 is offline   Trả Lời Với Trích Dẫn
Old 23-02-2012, 10:52 PM   #15
namdung
Administrator

 
Tham gia ngày: Feb 2009
Đến từ: Tp Hồ Chí Minh
Bài gởi: 1,343
Thanks: 209
Thanked 4,066 Times in 778 Posts
Gửi tin nhắn qua Yahoo chát tới namdung
Các bạn thân mến, như vậy là đã có kết quả sơ bộ VMO (dù là từ các nguồn thông tin rời rạc). Có lẽ chỉ ngày mai thôi là sẽ có kết quả chính thức. Trong số các thí sinh, sẽ có những người đạt giải, được tham dự kỳ thi TST, có thí sinh chỉ đạt giải ba, giải KK, và cũng có rất nhiều thí sinh không đạt giải. Đó cũng là chuyện bình thường trong bất kỳ một cuộc thi nào. Có bạn nào đó sẽ cảm thấy nuối tiếc vì đã không làm bài đúng phong độ, đã trình bày không thật tốt, đã sai sót trong tính toán. Tuy nhiên, tất cả rồi sẽ qua đi và những điều tốt đẹp nhất sẽ vẫn còn đọng lại. Ôn luyện cho VMO, các bạn đã học được nhiều kiến thức mới, được thử thách với những bài toán khó, được tư duy, được làm việc và được chuẩn bị cho những thách thức lớn lao hơn còn ở phía trước.

Vì thế, sau khi biết kết quả chính thức VMO, hãy tiếp tục làm tốt các công việc của mình. Các bạn lọt vào Top 42 sẽ chuẩn bị cho kỳ thi chọn đội tuyển. Các bạn còn lại (lớp 12) sẽ ôn thi tốt nghiệp và ôn thi ĐH, các bạn lớp 11 sẽ tìm cách giữ lửa để tiếp tục cho kỳ thi năm sau (hoặc một số bạn sẽ chọn một con đường khác). Các bạn đã cố gắng làm tốt các công việc đã qua, vậy thì hãy tiếp tục cố gắng với những công việc đang tới. Luôn luôn gắng sức và luôn hướng về phía trước. Đó là bí quyết đơn giản của những người thành công.
[RIGHT][I][B]Nguồn: MathScope.ORG[/B][/I][/RIGHT]
 
namdung is offline   Trả Lời Với Trích Dẫn
Trả lời Gởi Ðề Tài Mới

Bookmarks


Quuyền Hạn Của Bạn
You may not post new threads
You may not post replies
You may not post attachments
You may not edit your posts

BB code is Mở
Smilies đang Mở
[IMG] đang Mở
HTML đang Tắt

Chuyển đến


Múi giờ GMT. Hiện tại là 10:33 PM.


Powered by: vBulletin Copyright ©2000-2024, Jelsoft Enterprises Ltd.
Inactive Reminders By mathscope.org
[page compression: 127.05 k/143.84 k (11.67%)]